0 Daumen
470 Aufrufe

Aufgabe:

Die Folge (an)n∈N sei rekursiv definiert durch
\(a_1 = 3; a_{n+1} = 4 − \frac{4}{a_n} ~~~ (n\in\N)\)

Beweisen Sie mit vollständiger Induktion, dass an > 2 für alle n ∈ N ist.


Problem/Ansatz:

Hallo kann mir einer bei der aufgabe behilflich sein ?:)

Avatar von

Vom Duplikat:

Titel: Rekursiv definierte Folge

Stichworte: rekursiv

Aufgabe

Die Folge \( \left(a_{n}\right)_{n \in \mathbb{N}} \) sei rekursiv definiert durch
\( a_{1}=3, \quad a_{n+1}=4-\frac{4}{a_{n}} \quad(n \in \mathbb{N}) \)
(i) Beweisen Sie mit vollständiger Induktion, dass \( a_{n}>2 \) für alle \( n \in \mathbb{N} \) ist.
(ii) Zeigen Sie, dass die Folge \( \left(a_{n}\right)_{n \in \mathbb{N}} \) monoton fallend ist.
(iii) Folgern Sie, dass die Folge \( \left(a_{n}\right)_{n \in \mathbb{N}} \) konvergiert und bestimmen Sie den Grenzwert.

Vom Duplikat:

Titel: Welche Menge sind die und auch mit Begründung

Stichworte: mengen

Aufgabe
Die Folge \( \left(a_{n}\right)_{n \in \mathbb{N}} \) sei rekursiv definiert durch
\( a_{1}=3, \quad a_{n+1}=4-\frac{4}{a_{n}} \quad(n \in \mathbb{N}) \)
(i) Beweisen Sie mit vollständiger Induktion, dass \( a_{n}>2 \) für alle \( n \in \mathbb{N} \) ist.
(ii) Zeigen Sie, dass die Folge \( \left(a_{n}\right)_{n \in \mathbb{N}} \) monoton fallend ist.
(iii) Folgern Sie, dass die Folge \( \left(a_{n}\right)_{n \in \mathbb{N}} \) konvergiert und bestimmen Sie den Grenzwert.

2 Antworten

0 Daumen

(1.) Für n=1 ist an>2 wegen a1=3  und 3>2 offensichtlich erfüllt.

(2.)  Sei nun  n eine beliebige natürliche Zahl mit n≥1.  Wir dürfen dann voraussetzen ("Induktionsvoraussetzung"), dass  an>2  ist.

Dann ist   an+1 = 4 - 4/an größer als 2 , weil  4/an  wegen an>2  eine positive Zahl kleiner als 2 ist, und die Subtraktion  4 - (Zahl kleiner als 2) ergibt eben ein Resultat größer als 2.

Aus (1.) und (2.) zusammen ergibt sich nach dem Prinzip der vollständigen Induktion, dass  an>2  für alle  n∈ℕ .

Avatar von 3,9 k

hallo danke für die antwort :) die nummer 1 verstehe ich aber irgendwie wird mir die nummer 2 nicht ganz deutlich ich soll doch beweisen an grösser als die zahl 2 ist aber warum kann ich davon ausgehen das eine positive zahl kleine als 2 ist ? oder liege ich komplett falsch

Wenn  an>2 , ist an bestimmt auch erst mal positiv. Die Division 4 / an  ergibt dann ein Ergebnis, das ebenfalls positiv und zweitens  kleiner als  4 / 2 = 2  ist.

0 Daumen

Hallo
was kannst du nicht? Grenze? Monotonie? Grenzwert?
bitte stell nicht einfach Aufgaben mit komischen Titeln hier rein, sondern sag , was genau du nicht kannst.

Die Induktion in i) ist so einfach dass du sie sicher kannst,
ii) zeige durch nachrechne dass an+1-an<0
iii) für an und an+1  Grenzwert g einsetzen und daraus g bestimmen

Gruß lul

Avatar von 106 k 🚀

Ein anderes Problem?

Stell deine Frage

Willkommen bei der Mathelounge! Stell deine Frage einfach und kostenlos

x
Made by a lovely community